LSAT and Law School Admissions Forum

Get expert LSAT preparation and law school admissions advice from PowerScore Test Preparation.

 Administrator
PowerScore Staff
  • PowerScore Staff
  • Posts: 8916
  • Joined: Feb 02, 2011
|
#81367
Complete Question Explanation

The correct answer choice is (E).

Answer choice (A):

Answer choice (B):

Answer choice (C):

Answer choice (D):

Answer choice (E): This is the correct answer choice.

This explanation is still in progress. Please post any questions below!
 avengingangel
  • Posts: 275
  • Joined: Jun 14, 2016
|
#30941
I was between E and A, and chose E. I chose E because of the question posed in lines 31-33, and the distinction of Haffer's hypothesis as (seemingly) contrasted with Sanders', in lines 45-47. But I don't fully understand why A is wrong: it seems like lines 21-24 suggest that Sanders' hypothesis did in fact do what A is states... Also, I originally thought A was wrong because of "in the Amazon basin on the tropical rain forest" since in line 23 it says "climate of the Amazon basin has fluctuated..." But I'm not sure if that why, because Sanders' hypothesis is at times applied to the Amazon tropical rain forest (and not just the basin).

Also, what's is a basin ?? That's also what made this passage a little more difficult for me...and further confused me with "uplands" and "lowlands" -- I am not able to imagine them in my head (accurately) bc I don't know what they are. (And I know I don't NEED to know what everything is, which is evident by me getting all the questions correct! But I feel I got lucky here, bc I could have been asked a question where I really did need to know what "basin" means, and it's relationship to the rain forest, up/lowlands, etc. !!)

any further explanation? Thanks!
 David Boyle
PowerScore Staff
  • PowerScore Staff
  • Posts: 836
  • Joined: Jun 07, 2013
|
#31452
avengingangel wrote:I was between E and A, and chose E. I chose E because of the question posed in lines 31-33, and the distinction of Haffer's hypothesis as (seemingly) contrasted with Sanders', in lines 45-47. But I don't fully understand why A is wrong: it seems like lines 21-24 suggest that Sanders' hypothesis did in fact do what A is states... Also, I originally thought A was wrong because of "in the Amazon basin on the tropical rain forest" since in line 23 it says "climate of the Amazon basin has fluctuated..." But I'm not sure if that why, because Sanders' hypothesis is at times applied to the Amazon tropical rain forest (and not just the basin).

Also, what's is a basin ?? That's also what made this passage a little more difficult for me...and further confused me with "uplands" and "lowlands" -- I am not able to imagine them in my head (accurately) bc I don't know what they are. (And I know I don't NEED to know what everything is, which is evident by me getting all the questions correct! But I feel I got lucky here, bc I could have been asked a question where I really did need to know what "basin" means, and it's relationship to the rain forest, up/lowlands, etc. !!)

any further explanation? Thanks!

Hello,

The passage mentions "the absence of winter and glaciation", so answer A cannot really be right.
The rain forest is part of the whole Amazon basin.
One definition of "basin" is "a natural depression on the earth's surface, typically containing water"; another is "the tract of country that is drained by a river and its tributaries or drains into a lake or sea."

David

Get the most out of your LSAT Prep Plus subscription.

Analyze and track your performance with our Testing and Analytics Package.